Fundamantel

Ace your homework & exams now with Quizwiz!

The nursing process organizes your approach while delivering nursing care. To provide the best professional care to patients, nurses need to incorporate nursing process and:

critical thinking skills

According to Kohlberg, moral development is a component of psychosocial development. Moral development depends on the child's ability to integrate:

decisions of right and wrong

Which information supports the appropriateness of a nursing diagnosis?

defining characteristics

A nurse enters the room of a client and asks how they are doing. The client states, " I'm a little nervous this morning." What would be the nurse's best reply? a. "What is making you feel nervous?" b. "You do look as if you are feeling nervous." c. "Can I give you a backrub for your nerves?" d. "Can you tell more about how you are feeling?"

"Can you tell more about how you are feeling?"

As part of a faith community nursing program in her church, a nurse is developing a health promotion program on breast self-examination for the women's group. Which statement made by one of the participants is related to the individual's accurate perception of susceptibility to an illness?

"Since my mother had breast cancer, I know that I am at increased risk for developing breast cancer."

A patient is fearful of upcoming surgery and a possible cancer diag-nosis. He discusses his love for the Bible with his nurse, who recommends a favorite Bible verse. Another nurse tells the patient's nurse that there is no place in nursing for spiritual caring. What is the best response by the patient's nurse?

"Spiritual, mind, and body connections can affect health."

A 7-year-old child was recently diagnosed with asthma. A nurse is providing education to the child and her parents about the treatment and management of asthma and changes they need to make in their home environment to promote her health. Which statement made by the parents requires follow-up by the nurse?

"We haven't been spending time with our parents because we are so busy taking care of our daughter."

A patient tells the nurse, "My doctor told me to lose weight, exer-cise, stop smoking, and eat better. I am not sick at all. Why would he tell me that?" The nurse's best response would be:

"I believe he is trying to get you to think about ways that you can be healthier. All these things help to prevent future health problems."

The nurse is caring for a family where there is a strong family history of breast cancer. One of the family members says, "I am afraid Of having genetic testing. If it is positive, I know I have cancer." What is the nurse's best response?

"If the results are positive, it means you have a higher risk for breast cancer, not that you have cancer.

A novice nurse asks the preceptor to describe the primary purpose of evaluation. Which statement made by the nursing preceptor is most accurate? Correct!

"Nurses use evaluation to determine the effectiveness of nursing care."

A nurse contacts the health care provider about a change in a patient's condition and receives several new orders for the patient over the phone. When documenting telephone orders in the EHR, what should the nurse do?

"Read back" all telephone orders to the health care provider over the phone to verify all orders were heard, understood, and transcribed correctly before entering the orders in the EHR.

A 34-year-old single father who is anxious, tearful, and tired from caring for his three young children tells the nurse that he feels depressed and does not see how he can go on much longer. Which statement would be the nurse's best response?

"Tell me what you mean when you say you can't go on any longer."

A nursing student is giving a presentation to a group of other nursing students about the needs of patients with mental illnesses in the community. Which statement by the student indicates that the nursing professor needs to provide further teaching?

"The majority of patients with mental illnesses live in long-term care settings."

A client just received a diagnosis of cancer. Which statement by the nurse demonstrates empathy?

"This must be hard news to hear"

Based on the Transtheoretical Model of Change, what is the most appropriate response to a patient who states: "Me, stop smoking? I've been smoking since I was 16!"

"What do you think is the greatest reason why stopping smok-ing would be challenging for you?"

The order is Colace 150 mg by mouth once daily. The pharmacy sends the medication to the floor with the label: 150 mg/15 mL. How many milliliters will the nurse prepare?

15

The nurse is attempting to prompt the client to elaborate on reports of daytime fatigue. Which question should the nurse ask?

"what reasons do you think are contributing to your fatigue?"

The nurse recognizes that which factors influence a person's approach to death? (Select all that apply.)

1• Culture • Spirituality • Personal beliefs • Previous experiences with death

Place the steps of the EBP process in the appropriate order.

5. Cultivate a spirit of inquiry 2, Ask the clinical question in PICOT format. 4. Search for the most relevant and best evidence. 1. Critically appraise the evidence you gather 6. Integrate the evidence. 3. Evaluate the outcomes of the practice 7. Communicate the outcomes of the EBP change.

The nurse calculated 60% of a 72 year old males maximum heart rate is? Enter the number only. Round to the nearest whole number if necessary.

89

A nurse received change-of-shift reports on these four patients and starts rounding. Which patient does the nurse need to focus on as a priority?

A patient admitted yesterday with atrial fibrillation who now has a decreased level of consciousness

Intersectionality

A policy model that describes factors and power structures that shape and influence life

When assessing an older adult who is showing symptoms of anxiety, insomnia, anorexia, and mild confusion, what is the first assessment the nurse conducts?

A thorough physical assessment

While administering medications, a nurse realizes that a prescribed dose of a medication was not given. The nurse acts by completing an incident report and notifying the patient's health care provider. Which of the following is the nurse exercising?

Accountability

A nursing student is providing a hand-off report to the RN assuming the patient's care. The nursing student explains, "I ambulated him twice during the shift; he tolerated walking to end of hall and back each time with no shortness of breath. Heart rate was 88 and regular after exercise. The patient said he slept better last night after I closed his door and gave him a chance to have some uninterrupted sleep. I changed the dressing over his intravenous (IV) site and started a new bag of D‚½NS." Which intervention is a dependent intervention?

Administering IV fluids

Which example demonstrates a nurse performing the skill evaluation?

After completing the teaching, the nurse observes a patient draw up and administer an insulin injection.

Health disparities are unequal burdens of disease morbidity and mortality rates experienced by racial and ethnic groups. These disparities are often exacerbated by A. prejudice B. bias C. stereotyping D. All of the above

All of the above

Etic worldview

An outsider's perspective of an intercultural encounter.

A nurse is aware that preschoolers often display a developmental characteristic that makes them treat dolls or stuffed animals as if they have thoughts and feelings. This is an example of:

Animism

shared theory

Applies theory from other disciplines to nursing practice

A nurse is assessing an older adult brought to the emergency department following a fall and wrist fracture. The patient lives with her son, is very thin and unkempt, has a Stage 3 pressure injury on her coccyx, and has old bruising to the extremities in addition to her new bruises from the fall. She defers all of the questions to her caregiver son, who accompanied her to the hospital. What is the nurse's next step?

Ask the son to step out of the room so that she can complete her assessment.

Knowing

Asking a patient to talk about what the loss of a loved one means for the patient and the family.

An example of a nurse caring behavior that families of patients who are acutely ill perceive as important to patients' well-being is:

Asking permission before performing a procedure on a patient.

A patient has labored breathing, a respiratory rate of 28 per minute, and lung sounds that reveal wheezing bilaterally. The nurse starts an ordered intravenous infusion to administer medication that will relax the patient's airways. When the nurse asks how the patient feels, he responds by saying, "I feel as if I can breathe better? The nurse auscultates the patient's lungs and notes decreased wheezing with a respiratory rate of 22 per minute. Which of the following is a subjective evaluative measure?

Asking the patient to describe how his breathing feels

When designing a plan for pain management for a patient following surgery, the nurse assesses that the patient's priority is to be as free of pain as possible. The nurse and patient work together to identify a plan to manage the pain. The nurse continually reviews the plan with the patient to ensure that the patient's priority is met. If the nurse's actions are driven by respect for autonomy, what aspect of this scenario best demonstrates this action?

Asking the patient to establish the goal for pain control

A patient's plan of care includes the goal of increasing mobility this shift. As the patient is ambulating to the bathroom at the beginning of the shift, the patient falls. Which initial action will the nurse take next to most effectively revise the plan of care?

Assess the patient

King

Based on the belief that nurses should work with patients to develop goals for care

Orem

Based on the belief that people who participate in self-care activities are more likely to improve their health outcomes.

Neuman

Based on the theory that focuses on wellness and prevention of disease.

Specific

Be sure an outcome addresses only one patient behavior or response.

Contemporary nursing requires that the nurse have knowledge and skills for a variety of professional roles and responsibilities. Which of the following are examples of these roles and responsibilities? Select all that apply

Caregiver Autonomy Patient advocate Health promotion

Which is a person referring to when during an interview the person says, "I am a member of the sandwich generation?"

Cares for children and aging parents at the same time

A hospice nurse sits at the bedside of a male patient in the final stages of cancer. He and his parents made the decision that he would move home and they would help him in the final stages of his disease. The family participates in his care, but lately the nurse has increased the amount of time spent with the family. Whenever the nurse enters the room or approaches the patient to give care, the nurse touches his shoulder and states, "I am your nurse, and I am here for you." This is an example of what type of touch.

Caring touch

When documenting an assessment of a patient's cardiac system in an EHR, the nurse uses the computer mouse to select the "WNL" statement to document the following findings: "Heart sounds SI & S2 auscultated. Heart rate between 80 and 100 beats/min, and regular. Denies chest pain." This is an example of using which of the following documentation formats?

Charting by exception (CBE)

A nurse implements an EBP change that teaches patients the importance of taking their diabetes medications correctly and regularly on time using videos streamed on the Internet. The nurse measures the patients' behavioral outcome from the practice change by using which type of measurement?

Checking patients' blood sugars

The American Red Cross was founded by

Clara Barton

The nurse is administering a dose of metoprolol to a patient and is completing the steps of bar-code medication administration within the EH. As the bar-code information on the medication is scanned, an alert that states, "Do not administer dose if apical heart rate (HR) is <60 beats/min or systolic blood pressure (SP) is <90 mm Hg" appears on the computer screen. The alert that appeared on the computer screen is an example of what type of system?

Clinical decision support system (CDSS)

A home health nurse notices significant bruising on a 2-year-old patient's head, arms, abdomen, and legs. The patient's mother describes the patient's frequent falls. What is the best nursing action for the home health nurse to take?

Contact a child abuse hotline

A community health nurse conducts a community assessment focused on adolescent health behaviors. The nurse determines that a large number of adolescents smoke. Designing a smoking cessation program at the youth community center is an example of which nursing role?

Counselor

A nurse interviews and conducts a physical examination of a patient that includes the following findings: reduced range of motion of lower hip, reduced strength in left leg, and difficulty turning in bed without assistance. This data set is an example of:

Data cluster.

You are preparing a presentation for your classmates regarding the clinical care coordination conference for a patient with terminal cancer. As part of the preparation, you have your classmates read the Nursing Code of Ethics for Professional Registered Nurses. Your instructor asks the class why this document is important. Which statement best describes this code?

Defines the principles of right and wrong to provide patient care

A nurse is completing a health history with the daughter of a newly admitted patient who is confused and agitated. The daughter reports that her mother was diagnosed with Alzheimer disease 1 year ago but became extremely confused last evening and was hallucinating. She was unable to calm her, and her mother thought she was a stranger. On the basis of this history, the nurse suspects that the patient is experiencing:

Delirium

A patient who is having difficulty managing his diabetes mellitus responds to the news that his hemoglobin a1c a measure of blood sugar control over the past 90 days, has increased by say-ing, "The hemoglobin a1c is wrong. My blood sugar levels have been excellent for the last 6 months." Which defense mechanism is the patient using?

Denial

A nurse conducted an assessment of a new patient who came to the medical clinic. The patient is 82 years old and has had osteoarthritis for 10 years and diabetes mellitus for 20 years. He is alert but becomes easily distracted during the assessment. He recently moved to a new apartment, and his pet beagle died just 2 months ago. He is most likely experiencing:

Depression

Which action indicates a nurse is using critical thinking for implementation of nursing care to patients?

Determines whether an intervention is correct and appropriate for the given situation

You are working on a patient care unit and observe several other nurses who are not following the agency's policy when preparing and administering medications. When you ask them to explain why they are doing this, they state the new medication administration technology installed on the unit takes too much time to use, so they are using workarounds to get their patient medications administered on time. Which is the best action to take first?

Discuss the situation with the nursing manager.

A patient who is newly divorced states, "Although I'm really scared about what happens next to my children and myself, I know that divorce is not uncommon today? What term does the nurse use in the patient's medical record to describe the characteristic displayed in this statement?

Durability

A man who is homeless enters the emergency department seeking health care. The health care provider indicates that the patient needs to be transferred to the city hospital for care before assessing the patient. This action is most likely a violation of which of the following laws?

Emergency Medical Treatment and Active Labor Act (EMTALA)

The use of diagnostic reasoning involves a rigorous approach to clinical practice and demonstrates that critical thinking cannot be done:

haphazardly

Maintaining belief

Encouraging a patient or family to use resources and trust previous de-cisions that have helped to resolve issues.

A nurse is caring for a man who is recently retired and who appears withdrawn. He says he is "bored with life." The nurse helps this individual find meaning in life by:

Encouraging him to reflect on his relationships with others

A nurse in a community health clinic reviews screening results from students in a local high school during the most recent academic year. The nurse discovers a 10% increase in the number of positive tuberculosis (TB) skin tests when comparing these numbers to the previous year. The nurse contacts the school nurse and the director of the health department. Together they begin to expand their assessment to all students and employees of the school district. The community nurse was acting in which nursing role(s)? (Select all that apply.)

Epidemiologist Collaborator

A nurse assesses a 78-year-old patient who weighs 108.9 kg (240 lb) and is partially immobilized because of a stroke. The nurse turns the patient and finds that the skin over the sacrum is very red and the patient does not feel sensation in the area. The patient has had fecal incontinence on and off for the last two days. The nurse identifies the nursing diagnosis of Risk for Impaired Skin Integrity. Which of the following outcomes is appropriate for the patient?

Erythema of skin will be mild to none within 48 hours.

A critical care nurse is using a new research-based intervention to correctly position patients who are on ventilators to reduce pneumonia caused by accumulated respiratory secretions. This is an example of which QSEN competency?

Evidence-based practice

Clinical Nurse Specialist

Expert clinician in a specialized area of practice.

The application of deontology does not always resolve an ethical problem. Which of the following statements best explains one of the limitations of deontology?

In a diverse community it can be difficult to find agreement on which principles or rules are most important.

During a visit to a family clinic, a nurse teaches a mother about immunizations, the use of car seats, and home safety for an infant and toddler. Which type of nursing interventions are these?

health promotion

Worldview

Factor that shapes how people perceive others and how they relate to reality

A hospice nurse is caring for a family that is providing end-of-life care for their grandmother, who has terminal breast cancer. The nurse focuses on symptom management for the grandmother and on helping the family with developing coping skills. This approach is an example of which of the following?

Family as patient

A woman has severe life-threatening injuries, is unresponsive, and is hemorrhaging following a car accident. The health care provider ordered 2 units of packed red blood cells to treat the woman's anemia. The woman's husband refuses to allow the nurse to give his wife the blood for religious reasons. What is the nurse's responsibility?

Gather more information about the wife's preferences and determine whether the husband has her power of attorney for health care.

A 50-year-old woman has elevated serum cholesterol levels that increase her risk for cardiovascular disease. The nurse helps this patient control this risk factor by assessing the patient's current diet trends and describing dietary changes to reduce the patient's risk for developing cardiovascular disease. This nursing activity is d form of:

Health education

Which task is appropriate for a registered nurse (RN) to delegate to an AP?

Helping the patient to the bathroom before leaving for the operating room

A nurse's assessment reveals a patient having frequent voiding and pain when she urinates. Her body temperature is 38°C (100.4°F). The nurse asks whether she has to go to the bathroom at night, and the patient responds,' "Yes." When asked how often, the patient replies, "About three times a night.? The nurse asks if having to urinate at night is recent or normal for the patient. The patient explains, "I usually go once a night but that is all? The nurse then asks, "When you feel the need to go, can you reach the toilet in time?" The patient says, "Oh, yes, I can." The nurse asks, "And have you had any leaking of urine?" The patient denies leaking. When asked if she is having any back or abdominal pain, the patient denies discomfort. The nurse then gathers a urine specimen from the patient and inspects its char-acter, noting it is cloudy and foul smelling. Which of the following nursing diagnoses are indicated by cues in this patient's assessment?

Impaired Urination

Timed

Include when an outcome is to be met.

Which principle is most important for a nurse to follow when using a clinical practice guideline for an assigned patient?

Individualizing how to apply the clinical guideline for a patient

During a nursing assessment a patient displayed several behaviors. Which behavior suggests the patient may have a health literacy problem?

Patient has difficulty completing a registration form at a medical office

Emic worldview

Insider's perspective in an intercultural encounter

A nurse is caring for a patient newly diagnosed with testicular cancer. He asked the nurse to help him find the meaning of cancer by supporting beliefs about life. This is an example of

Instilling hope and faith.

The nurse is reviewing health care provider orders that were handwritten on paper when all computers were down during a system upgrade. Which of the following orders contains an inappropriate abbreviation included on TIC's "do not use" list and should be clarified with the health care provider?

Insulin aspart 8u SQ every morning before breakfast

Which explanation provided by the nurse is the most accurate meaning for "providing culturally congruent care"?

It fits the patient's valued life patterns and set of meanings.

Dave reports being happy and satisfied with his life. What do we know about him?

It is difficult to determine Dave's developmental stage since most people report overall satisfaction with their lives in all stages.

A nurse desires to communicate with a young woman who is Serbian and who has limited experience with being in a hospital. The nurse has 10 years of experience caring for Serbian women. The patient was admitted for a serious pregnancy complication. Apply the LEARN model and match the nurse's behaviors with each step of the model. 1. L ___________ 2. E ___________ 3. A ___________ 4. R __________ 5. N __________

L- The nurse attends to the patient and listens to her story about hospitals in Serbia. E- The nurse shares her perception of the woman's experiences as a patient. A- The nurse notes that she has learned that fathers can visit mothers at any time in both Serbia and the United States. R- The nurse involves the patient in a discussion of the treatment options for her condition. N- The nurse asks the patient how she can maintain bed rest when she returns home.

A nurse is analyzing data generated during a patient assessment to determine the best plan of care. Which layer of the NCSBN-CIM best describes the cognitive process being used by the nurse at this time?

Layer 3

What factor results in vulnerable populations being more likely to develop health problems?

Limited access to health care services

Which of the following statements is true regarding Magnet status recognition for a hospital?

Magnet is a special designation for hospitals that achieve excellence in nursing practice.

A nurse completes an Incident Report after a client falls while getting out of bed unassisted. Which is the purpose of this report?

Make data available for quality control analysis.

The nurses on a medical unit have seen an increase in the number of pressure injuries developing in their patients. The nurses decide to initiate a PI project using the PDSA model. Which of the following is an example of "Plan" from that model?

Meeting with all disciplines to develop a multidisciplinary approach for reducing pressure injuries.

A student is participating in a postclinical conference with the other students in the clinical group and an instructor. The student states, "My patient has two nursing diagnoses I chose to focus on, Risk for Impaired Skin Integrity and Lack of Knowledge regarding diabetic diet restrictions. I observed no pressure areas this morning. Because of her weight (100 kg; 220 lb), I turned her every two hours, and we put her on a pressure-relieving surface. We discussed how diabetes mellitus affects her circulation. During the day she had a glucose tolerance test that was normal. We discussed her diet during lunchtime. She completed a menu for a day with food choices that fit her diet. Before the shift was over, I talked with her physician about the patient's medication plan; her blood glucose levels have been higher than desired." Which of the following taken from the student's summary is a nurse-sensitive outcome?

Menu completion with food choices fitting diet

Health care reform will bring changes in the emphasis of care. Which of these models is expected from health care reform?

Moving from an acute illness to a health promotion, illness prevention model.

Attainable

Mutually set an outcome a patient agrees to meet.

Your patient is about to undergo a controversial orthopedic procedure. The procedure may cause periods of pain. Although nurses agree to do no harm, this procedure may be the patient's only treatment choice. This example describes the ethical principle of:

Nonmaleficence

When taking care of patients, a nurse routinely asks whether they take any vitamins or herbal medications, encourages family members to bring in music that the patients like to help the patients relax, and frequently prays with her patients if that is important to them. The nurse is practicing which model?

holistic

The nurses on an acute care medical floor notice an increase in pressure injury formation in their patients. A nurse consultant decides to compare two types of treatment. The first is the procedure currently used to assess for pressure injury risk. The second uses a new assessment instrument to identify at-risk patients. Given this information, the nurse exemplifies which career?

Nurse researcher

Which of the following statements related to theory-based nursing practice are correct? (Select all that apply.)

Nursing theory differentiates nursing from other disciplines. Integrating theory into practice promotes coordinated care delivery. Nursing knowledge is generated by theory. Evidence-based practice results from theory-testing research.

When Ryan was 3 months old, he had a toy train; when his view of the train was blocked, he did not search for it. Now that he is 9 months old, he looks for it, reflecting the presence of.

Object permanence

A nurse working the evening shift has five patients and is teamed up with an assistive personnel. One of the assigned patients has just returned from surgery, one is newly admitted, and one has requested a pain medication. The patient who has returned from surgery just minutes ago has a large abdominal dressing, is still on oxygen by nasal cannula, and has an intravenous line. One of the other patients has just called out for assistance in setting up a meal tray. Another patient is stable and resting comfortably. Which patient is the nurse's current greatest priority?

Patient who returned from surgery

In preparing to collect a nursing history for a patient admitted for elective surgery, which of the following data are part of the review of present illness in the nursing health history?

Patient's explanation for what might be the cause of symptoms that require surgery

A nurse is caring for a patient admitted to the neurological with the diagnosis of a stroke and right-sided weakness. The nurse assumes responsibility for bathing and feeding the patient until the patient can begin performing these activities. The nurse in situation is applying the theory developed by:

Orem

Many older homes in a neighborhood are undergoing a lot of restora-tion. Lead paint was used to paint the homes when they were built. The community clinic in the neighborhood is initiating a lead screening program. This activity is based on which social determinant of health?

Physical environment

Nurse anesthetist

Plans and delivers anesthesia and pain management to patients across the life span.

A nurse received a bedside report at the change of shift with the night-shift nurse and the patient. The nursing student assigned to the patient asks to review the patient's medical record. The nurse lists patients' medical diagnoses on the message boards in the patients' rooms. Later in the day the nurse discusses the plan of care for a patient who is dying with the patient's family. Which of these actions describes a violation of HIPAA?

Posting medical information about the patient on a message board in the patient's room

When preparing a 4-year-old child for a procedure, which method is developmentally most appropriate for the nurse to use?

Preparing the child through play with a doll and toy medical equipment

A nurse who works in an outpatient chemotherapy infusion center is assigned to the care of a 56-year-old male patient who is receiving chemotherapy for colon cancer. This is the patient's first clinic visit. The nurse reviews the patient's medical record and sees a note about the patient receiving instruction on how chemotherapy treats cancer; the note does not summarize the patient's response. The nurse's assessment reveals that the patient is motivated to learn more about his chemotherapy and is alert and currently feeling well. The nurse has prepared the first infusion, regulated it, and now takes time to begin instruction. Which teaching approaches are best suited for this situation?

Provide verbal one-on-one instruction, with the patient participating in selection of content.

The examination for RN licensure is the same in every state in the United States. This examination:

Provides a minimal standard of knowledge for an RN in practice

Nurse Practitioner

Provides comprehensive care, usually in a primary care setting, directly managing the medical care of patients who are healthy or who have chronic conditions.

Nurse Midwife

Provides independent care, including pregnancy and gynecological services.

Doing for

Providing an opportunity for a patient's specific religious practice as the patient would do it.

A 55-year-old man has been in the hospital for over a week following surgical complications. The patient has had limited activity but is now ordered to begin a mobility program. The patient just returned from several diagnostic tests and tells the nurse he is feeling fatigued. The nurse prepares to instruct the patient on the mobility program protocol. Which of the following learning principles will likely be affected by this patient's condition?

Readiness to learn

A young woman comes to a clinic for the first time for a gynecological examination. She asks about the procedure, who will be in the room with her, and does she have to remove all her clothes. Which nursing behavior applies Swanson's caring process of "knowing" the patient?

Recognizing that the patient is modest and maintaining her privacy during the examination.

A nurse is using Healthy People 2030 to establish goals for the community. Which goal is priority? a. reduce health care cost b. increase life expectancy c. provide services close to where clients live d. isolate clients to prevent the spread of disease

increase life expectancy

The nurse is caring for a patient with an abdominal obstruction. The nurse irrigates the patient's nasogastric tube and reports the amount of fluid aspirated from the patient's stomach to the health care provides. The patient has an IV infusing; the nurse changes the transparent dressing over the IV site and instructs the patient to report any tenderness at the site. Which of these interventions is an indirect care measure?

Report of the amount of fluid aspirated

A nurse teaches parents how to have their children learn impulse control and cooperative behaviors. This would be during which of Erikson's stages of development?

initiative versus guilt

A family includes a mother, a stepfather, two teenage biological daughters of the mother, and a biological daughter of the father. The father's daughter just moved home following the loss of her job in another city. The family is converting a study into a bedroom and is in the process of distributing household chores. Nursing assessment reveals that all members of the family think that their family can adjust to lifestyle changes. This is an example of family:

Resiliency

A patient recovering from open heart surgery is taught how to cough and deep breathe using a pillow to support or splint the chest incision. Following the teaching session, which of the fol. lowing is the best way for the nurse to evaluate whether learning has taken place?

Return demonstration

The nurse finds it difficult to care for a patient whose advance directive states that no extraordinary resuscitation measures should be taken. Which step may help the nurse to find resolution in this assignment?

Review one's own personal values

A home health nurse completes a home safety assessment. The data reveal that the patient is 71 years old with bilateral cataracts that are causing blurred vision and sensitivity to bright lights. The patient is recovering from a stroke that has caused left-sided weakness of the leg, with an unsteady gait. When talking with the patient, the nurse learns the patient lives alone and fell in the bathroom 4 months ago. Rooms in the home are in disrepair. The nurse identifies nursing diagnoses of Impaired Vision, Impaired Mobility, and Risk for Fall. Which diagnosis is the nurse's priority?

Risk for fall

A patient in the ICU experiences a sentinel event related to central line catheter care that resulted in serious injury. What PI model should the unit use to identify errors that led to the sentinel event?

Root cause analysis

A nurse is caring for a patient who recently lost a leg in a motor vehicle accident. The nurse best assists the patient to cope with this situation by applying which of the following theories?

Roy

Elizabeth, who is having unprotected sex with her boyfriend, comments to her friends, "Did you hear about Kathy? You know, she fools around so much; I heard she was pregnant. That would never happen to me!" This is an example of adolescent:

Sense of invulnerability.

Realistic

Set an outcome that a patient can meet based upon physiological, emotional, economic, and sociocultural resources.

A nurse is preparing to teach a patient who has sleep apnea how to use a continuous positive airway pressure (CAP) machine at night. Which action is most appropriate for the nurse to perform first?

Set mutual goals for the education session.

practice theory

Specific to a particular situation; brings theory to the bedside

A nurse is planning care for a patient going to surgery. Who is responsible for informing the patient about the surgery along with possible risks, complications, and benefits?

Surgeon

Being with

Telling a family that you understand their anger about the side effects of chemotherapy.

A patient suddenly experienced a severe headache with numbness and decreased movement in the left arm. An emergency brain scan confirmed a cerebral vessel clot. With a stroke confirmed, the emergency room physician consults with a neurosurgeon to schedule an emergent angiogram to remove the clot. Which teaching approach is most appropriate for explaining to the patient what to expect from the procedure?

Telling approach

A patient discharged a week ago following a stroke is currently participating in rehabilitation sessions provided by nurses, physical therapists, and registered dietitians in an outpatient setting. In what level of prevention is the patient participating?

Tertiary prevention

The nurse is caring for a patient who is near death. What assessment finding cues the nurse that death is approaching?

mottling

Which of the family caregivers listed below will the nurse expect to be most at risk for experiencing poor health outcomes?

The 68-year-old spouse of a patient who is experiencing worsening dementia.

A nurse ensures that each patient's room is clean; well ventilated; and free from clutter, excessive noise, and extremes in temperature. Which theorist's work is the nurse practicing in this example?

nightingale

Advocacy

patient at the end of life wants to go home to die, but the family wants every care possible. The nurse contacts the primary care provider about the patient's request.

A nurse in the recovery room is monitoring a patient who had a lef knee replacement. The patient arrived in recovery 1 hour ago. The nurse observes the patient to be restless, turning frequently, and groaning; the patient's heart rate is 92 compared with 76 immediately postoperatively. Blood pressure is stable since admission to the recovery room. The nurse reviews the medical orders for analgesic therapy. The nurse notes that the postoperative dose of an ordered analgesic has not yet been given. Which of the following factors is most likely to cause the nurse to reflect on the patient's situation?

The delay in administration of the analgesic

The nurse is caring for a patient who needs a liver transplant to survive. This patient has been out of work for several months, does not have health insurance, and cannot afford the procedure. Which of the following statements speaks to the ethical elements of this case?

The health care team should select a plan that considers the principle of justice as it pertains to the distribution of health care resources.

A nurse is reading a research article discussing a new practice to decrease the incidence of catheter-associated urinary tract infections. One section of the article describes who was studied and how the data were collected to answer the research questions and hypotheses.

The methods

Cultural desire

The motivation of a health care professional to "want to" engage in cultural competence

A nurse enters a patient's room and begins a conversation. During this time the nurse evaluates how a patient is tolerating a new diet plan. The nurse decides to also evaluate the patient's expectations of care. Which of the following is appropriate for evaluating a patient's expectations of care?

The nurse asks, "Do you believe that you received the information you needed to follow your diet?"

Match the components of PICO using the question "Does the use of guided imagery compared with standard care affect the postoperative pain in hospitalized adolescents?

patient/population -Hospitalized adolescents intervention- Guided imagery comparison- Adolescents receiving outcome- postoperative pain

The components of the nursing metaparadigm include:

person, health, environment, nursing

What outcome demonstrates the effective use of silence as a therapeutic communication technique?

The patient perceives having gained insight into the issue after the conversation.

Resolution of an ethical problem involves discussion with the patient, the patient's family, and participants from appropriate health care disciplines. Which statement best describes the role of the nurse in the resolution of ethical problems?

To articulate the nurse's unique point of view, including knowledge based on clinical and psychosocial observations.

Measurable

Use a term in an outcome statement that allows for observation as to whether a change takes place in a patient's status.

Nurses in a community clinic are conducting an EBP project focused on improving the outcomes of children with asthma. The PICO question asked by the nurses is "In school-age children, does the use of an electronic gaming education module versus educational book affect the usage of inhalers?" In the question, what is the "O"?

Use of inhalers

A nurse is caring for a patient with end-stage lung disease. The patient wants to go home on oxygen and be comfortable. The family wants the patient to have a new surgical procedure. The nurse explains the risk and benefits of the operation to the family and discusses the patient's wishes with them. The nurse is acting as the patient's:

advocate

A nurse assesses a client has slurred speech and a retained bolus of food in the mouth. The presence of which additional client assessments should be clustered with the group of signs and symptoms? Select all that apply a. dyspepsia b. coughing c. drooling d. gurgling e. plaque

b. coughing c. drooling d. gurgling

grand theory

Very abstract; attempts to describe nursing in a global context

Henderson

based on 14 activities, the belief that the nurse should assist patients with meeting needs until they are able to do so independently

A nurse working in a large occupational health clinic knows that many of the workers at her company are marginalized and at risk for poor health outcomes. Which of the following individuals are most likely to be marginalized?

Workers who have a high school education

The nurse asks a patient the following series of questions: "Describe for me how much you exercise each day."How do you tolerate the exercise?" "Is the amount of exercise you get each day the same, less, or more than what you did a year ago?" This series of questions would likely occur during which phase of a patient-centered interview?

Working phase

A nursing student is working with a faculty member to identify a nursing diagnosis for an assigned patient. The student has assessed that the patient is undergoing radiation treatment to the abdomen, has liquid stool, and the skin is clean and intact. The student selects the nursing diagnosis Impaired Skin Integrity. The faculty member explains that the student has made a diagnostic error for which of the following reasons?

Wrong diagnosis

Accountability

You administer a once-a-day cardiac medication at the wrong time, but nobody sees it.

Confidentiality

You see an open medical record on the computer and close it so that no one else can read the record without proper access.

responsibility

You tell your patient that you will return in 30 minutes to give him his next pain medication.

If a nurse decides to withhold a medication because it might further lower the patient's blood pressure, the nurse will be practicing the principle of:

accountability

Middle Range Theory

address specific phenomena and reflect practice

A nurse is conducting a home visit with a new mom and her three children. While in the home the nurse weighs each family member and reviews their 3-day food diary. She checks the mom's blood pressure and encourages the mom to take the children for a 30-minute walk every day. The nurse is addressing which level of need, according to Maslow's Hierarchy of Needs?

physiological

Consultation occurs most often during which phase of the nursing process?

planning

In planning care for the older adult with dementia, the nurse identifies which intervention as the first priority goal of care?

prevent injury

A travel nurse has taken an assignment at a health care facility where nurses assume responsibility for a caseload of patients over a period. This type of nursing exemplifies:

primary nursing.

A hospice client is clearly dehydrated and the family is arguing over whether or not the client should receive intravenous (IV) fluids. The nurse would guide this discussion based on what knowledge about dehydration in the terminally ill client? a. if the terminally ill client complains of thirst, he/she is dehydrated b. peripheral edema in the terminally ill client indicates fluid overload c. the emphasis on all treatments should be on comfort and reduction of symptoms d. the only choices for hydration are oral and intravenous

the emphasis on all treatments should be on comfort and reduction of symptoms

Which of the following statements correctly describes the evaluation process? (Select all that apply.)

• Evaluation involves reflection on the approach to care. • Evaluation involves making clinical decisions. • Evaluation requires the use of assessment skills.

To best assist a patient in the grieving process, which factors are most important for the nurse to assess? (Sclect all that apply.)

• Previous experiences with grief and loss • Religious affiliation and denomination • Ethnic background and cultural practices

A 10-year-old girl was playing on a slide at a playground during a summer camp. She fell and broke her arm. The camp notified the parents and took the child to the emergency department according to the camp protocol for injuries. The parents arrive at the emergency department and are stressed and frantic. The 10-year-old is happy in the treatment room, eating a Popsicle and picking out the color of her cast. List in order of priority what the nurse should say to the parents.

• your daughter is happy in the treatment room, eating a popsicle & picking out the color of her cast." • I want to be sure you are ok. Let's talk about what your concerns are about your daughter before we go see her." • I'll have the doctor come out & talk to you as soon as possible." • can I contact someone to help you?"

A nurse is conducting a patient-centered interview. Place the state-ments from the interview in the correct order, beginning with the first statement a nurse would ask.

•"My name is Terry. I'lI be the nurse taking care of you today." • "Tell me what brought you to the hospital." • "You say you've lost weight. Tell me how much weight you've lost in the past month." • "So, to summarize, you've lost about 6 pounds in the past • "I have no further questions. Is there anything else you wish to ask me?"

Which techniques demonstrate a therapeutic response to an adult patient who is anxious? (Select all that apply.)

•Providing good eye contact •Demonstrating a calm presence •Spending time attentively with the patient

Review the following nursing diagnoses and identify the diagnoses that are stated correctly. (Select all that apply.)

•Risk for Infection • Activity Intolerance related to physical deconditioning evidenced by exertional dyspnea

Review the following nursing diagnoses that are stated correctly. (Select all that apply). 1. Offer frequent skin care because of Impaired Skin Integrity. 2. Risk for Infection. 3. Chronic Pain related to osteoarthritis evidenced by reduced hip range of motion. 4. Activity Intolerance related to physical deconditioning evidenced by exertional dyspnea. 5. Lack of knowledge related to laser surgery.

•Risk for Infection •Activity Intolerance related to physical deconditioning evidenced by exertional dyspnea

The physician orders dilaudid syrup 7.5 mg by mouth every four hours for pain. The pharmacy fills the clients prescription with dilaudid syrup 20 mg/ml. The nurse will administer how many milliliters (ml) per dose for pain?

0.38

The nurse works at an agency where military time is used for documentation and needs to document that a patient was medicated for pain after midnight. Identify the correct military time to document medication administered at 12:05 a.m.

0005

The nurse is changing the dressing over the midline incision of a patient who had surgery. Assessment of the incision reveals changes from what was documented by the previous nurse. After documenting the current wound assessment, the nurse contacts the surgeon by telephone to discuss changes in the incision that are of concern. Which of the following illustrates the most appropriate way for the nurse to document this conversation?

09-3-21 (1015): Surgeon contacted by phone. Notified about new area of bright red erythema extending approximately 1 inch around circumference of midline abdominal incision and oral temperature of 101.5 F. No orders received. T. Wright, RN

The public health nurse is working with the county health department on a task force to fully integrate the goals of Healthy People 2030. Most of the immigrant population do not have a primary care provider, nor do they participate in health promotion activi-ties; the unemployment rate in the community is 25%. How does the nurse determine which goals need to be included or updated? (Select all that apply.)

1. Assess the health care resources within the community. 2. Assess the existing health care programs offered by the county health department. 3. Compare existing resources and programs with Healthy People 2030 goals.

A community health nurse is working in a clinic with a focus on asthma and allergies. What is the primary focus of the community health nurse in this clinic setting? (Select all that apply.)

1. Decrease the incidence of asthma attacks in the community. 2. Increase patients' ability to self-manage their asthma. 3. Provide asthma education programs for the teachers in the local schools.

The public health nurse is working with the local city/county health department during a pandemic that has created a crisis within the community. What are responsibilities of the public health nurse during the pandemic? (Select all that apply.)

1. Educate the public on disease prevention 2. Serve as liaison between patients and health care services and providers 3. Investigate cases as they arise 4. Monitor trends of the disease outbreak 5. Assist with testing for identification of the disease

A nurse caring for a Bosnian community identifies that the children are under vaccinated and the community is and determines that there is a health clinic within a 5-mile radius. The nurse meets with the community leaders and explains the need for immunizations, the location of the clinic, and the process of accessing health care resources. what practices is the nurse providing?

1. Raising awareness about community resources for the children 2. Teaching the community about health promotion and illness prevention

The nurse in a new community-based clinic is requested to complete a community assessment. Order the steps for completing this assessment. 1. Structure or locale 2. Social systems 3. Population

1. Structure or locale 2. Population 3. Social systems

The provider orders digoxin 300 mcg intravenous (IV) bolus. The pharmacy sends digoxin 0.25 mg/ml. The nurse should administer how many milliliters (ml) for the IV bolus? Enter the number only (example- 0.8 is correct; eight is incorrect, 0.8 ml is incorrect). Round to one decimal point (tenths) if needed

1.2

Using Maslow's hierarchy of needs, identify the priority for a patient who is experiencing chest pain and difficulty breathing. 1. Self-actualization 2. Air, water, and nutrition 3. Safety 4. Esteem and self-esteem needs

2. Air, water, and nutrition

The nurse is discussing the advantages of using CPOE with a nursing colleague. Which statement best describes the major advantage of a CPOE system within an EHR?

CPOE improves patient safety by reducing transcription errors.

A nurse observes a patient walking down the hall with a shuffling gait. When the patient returns to bed, the nurse checks the strength in both of the patient's legs. The nurse applies the information gained to suspect that the patient has a mobility problem. This conclusion is an example of:

Clinical inference.

A health system upgraded its electronic health record across all its practice settings to enhance patient care and communication among health care providers. This is an example of which component of the Chronic Care Model?

Clinical information systems

The nurse spends time with the patient and family reviewing the dressing change procedure for the patient's wound. The patient's spouse demonstrates how to change the dressing. The nurse is acting in which professional role?

educator

A mother is concerned about her child's flulike symptoms. You learn from the health assessment that the mother practices use of "hot" and "cold" foods to treat ailments. Which of the following foods do you expect the mother to use to treat her child?

eggs

A nurse is assessing the family unit to determine their ability to adapt to the change required of a member having surgery. Which area is the nurse monitoring?

family resiliency

The nurse manager meets with the registered nursing staff about an increase in urinary tract infections in patients with a Foley catheter. The staff work together to review the literature on catheter-associated urinary tract infections (CAUTIs), identifies at-risk patients, and establishes new catheter care practices. This is an example of which QSEN competency?

quality improvement

A 34-year-old female executive has a job with frequent deadlines. She notes that when the deadlines appear, she tends to eat high-fat, high-carbohydrate foods. She also explains that she gets frequent headaches and stomach pain during these deadlines. After receiving health education from the nurse, the executive decides to try yoga. In this scenario yoga is best described as which one of the following?

stress management technique

A patient states, "I don't have confidence in my doctor. She looks so young." What is the nurse's therapeutic response?

tell me more about your concern

A patient is suffering from shortness of breath. The correct goal statement would be written as:

the patient will breathe unlabored at 14 to 18 breaths per minute by the end of the shift.

Which statements made by a nursing student about the teach-back technique show understanding of the technique? (Select all that apply.)

• "After teaching a patient how to use an inhaler, I need to use the teach-back technique to test my patient's ability to use the inhaler correctly. • "The teach-back technique is an ongoing process of asking patients for feedback." • "Using teach-back will help me identify explanations and communication strategies that my patients will most commonly understand." • "Using pictures, drawings, and models can enhance the effectiveness of the teach-back technique." • "When doing my patient teaching, I will use plain language to make the material easier to understand for the patient."

A married couple has three children. The youngest child just graduated from college and is moving to a different city to take a job. The other two children left home several years ago. The parents of both spouses are older and are beginning to need help to maintain their home. What assessment questions will help the nurse determine the family's functioning? (Select all that apply.)

• "Describe a recent family conflict and how your family resolved it." • "What coping strategies do you typically use as a family?" • "Who is involved in helping care for your parents?"

Which comments made by a patient show an understanding of the teaching on palliative care completed by the nurse?

• "Even though I'm continuing treatment, palliative care can help manage my symptoms and improve my quality of life.' • "Palliative care is appropriate for people with any diagnosis. • "Children are able to receive palliative care."

A nurse working on a medical patient care unit states, "I am having trouble sleeping, and I eat nonstop when I get home. All I can think of when I get to work is how I can't wait for my shift to be over. I wish i felt happy again." What are the best responses from the nurse manager? (select all that apply)

• "I'm sure this is just a phase you are going through. Hang in there. You'll feel better soon." • "I know several nurses who feel this way every now and then. Tell me about the patients you have cared for recently. Did you find it difficult to care for them?" • "The hospital just started a group where nurses get together to talk about their feelings. Would you like for me to e-mail the schedule to you?"

During a home visit, a patient states, "I am really upset about my heart failure. I can't go out to eat anymore with my friends, I have no energy, and I don't even want to talk on the phone. All I do is focus on how this disease has changed my life and how much time I have left to live" How should the nurse respond? (Select all that apply.)

• "Let's talk about going out to lunch. What is making you hesitant about eating with your friends?" • "Tell me about what types of activities you were doing before you knew you had heart failure." • "How has your heart failure affected your energy level?"

A nurse research investigator is explaining a research study to a patient. The patient is interested in participating in the research study. The nurse investigator reviews the informed consent with the patient. The patient asks the nurse why he must sign the consent to participate in the study. What is the nurse's best response (s) to the patient's question? (Select all that apply.)

• "The informed consent (IC) provides you with complete information on the research study." • "By reviewing the consent with you, I can make sure that you understand the study and what it means for you to par-ticipate." • "The informed consent provides your voluntary agreement to participate in the research study." • "The informed consent explains how the data collected from you will be kept confidential during and after the study."

A family consisting of a grandparent, two adults, and three school. age children just immigrated to the United States. They come to a community wellness center to establish health care. Which of the following questions does the nurse ask to assess the family's function? (Select all that apply.)

• "What does your family do to keep members healthy?" • "How does your family usually make decisions?" • "Which rituals or celebrations are important for your family?"

Which of the following are examples of the conventional reasoning form of cognitive development? (Select all that apply.)

• A 35-year-old woman is speaking with you about her recent diagnosis of a chronic illness. She is concerned about her treatment options in relation to her ability to continue to care for her family. As she considers the options and alternatives, she incorporates information, her values, and emotions to decide which plan will be the best fit for her. • A young father is considering whether or not to return to school for a graduate degree. He considers the impact the time commitment may have on the needs of his wife and infant son.

You are caring for a young female patient who is currently home-less. You believe she is experiencing IPV. Your assessment of the patient reveals several socioeconomic factors. Which ones are linked to IPV? (Select all that apply.)

• History of smoking marijuana • Recent divorce • High school education • Unemployment

The nurse identifies which of the following as outcomes measure-ments? (Select all that apply.)

• A nurse teaches a patient how to administer an injection and then watches the patient do a return demonstration. • A nursing unit adopts a set of strategies for reducing pressure injuries, and the UPC members use direct observation of the skin to measure incidence of pressure injuries. • A nursing unit implements a new fall prevention protocol and checks the monthly performance data for incidence of falls on the unit.

Which of the following scenarios demonstrate that patient learning has taken place? (Select all that apply.)

• A patient describes how to set up a pill organizer for newly ordered medicines. • A patient demonstrates how to take his blood pressure at home.

A nurse is seeing all of these patients in the community health clinic. The nurse identifies which patients) as having a health dis-parity? (Select all that apply.)

• A patient who has a homosexual sexual preference • A patient unable to access primary care services • A patient living with chronic schizophrenia

A patient's family member is considering having her mother placed in a nursing center. The nurse has talked with the family before and knows that this is a difficult decision. Which of the following criteria does the nurse recommend in choosing a nursing center? (Select all that apply.)

• Adequate staffing is available on all shifts • Social activities are available for all residents. • Staff encourage family involvement in care planning and assisting with physical care.

Which interventions does a nurse implement to help a patient at the end of life maintain autonomy while in a hospital? (Select all that apply.)

• Allow the patient to determine timing and scheduling of interventions. • Allow patients to have visitors at any time. •Encourage the patient to eat whenever he or she is hungry.

A nurse is conferring with another nurse about the care of a patient with a Stage 2 pressure injury. The two decide to review the clinical practice guideline of the hospital for pressure injury care. The use of a clinical practice guideline achieves which of the following? (Select all that apply.)

• Allows nurses to act more quickly and appropriately • Sets a level of clinical excellence for practice • Incorporates evidence- based interventions for stage Il pressure injury

Which nursing activities would help a patient self-manage rheumatoid arthritis? (Select all that apply).

• Assess the patient's beliefs about rheumatoid arthritis and its effects on activity levels. • Refer the patient to a support group that helps people with rheumatoid arthritis manage their disease.

A nurse is caring for a 66-year-old patient who lives alone and is receiving chemotherapy and radiation for a new cancer diagnosis. He is unable to care for himself because of severe pain and fatigue. He moves into his 68-year-old brother's home so his brother can help care for him. Which assessment findings indicate that this family caregiving situation will be successful? (Select all that apply.)

• Both the patient and his brother attend church together regularly. • Other siblings live in the same city and are willing to help. • The patient and his brother have a close network of friends.

When working with an older adult who is hearing impaired, the use of which techniques would improve communication? (Select al that apply.)

• Check for needed adaptive equipment. • Give the patient time to respond to questions. • Keep communication short and to the point.

While receiving a hand-off report, a nurse asks how a patient's condition from a serious infection changed since yesterday. The nurse leaving the shift reports that the patient has two priority nursing diagnoses fluid imbalance and fever. The receiving nurse begins to provide care by measuring the patient's body temperature, inspecting the condition of the skin, reviewing the intake and output record, and checking the summary notes describing the patient's progress since the day before. The nurse asks an assistive personnel to measure intake and output during the shift. What critical thinking indicators reflect the nurse's ability to perform evaluation? (Select all that apply.)

• Checking the summary notes • Asking the leaving RN about the patient's condition • Comparing current outcomes with those set for the patients goals • Reflecting on patient's progress

Which measures does a nurse follow when being asked to perform an unfamiliar procedure? (Select all that apply.)

• Checks scientific literature or policy and procedure • Determines whether additional assistance is needed • Collects all necessary equipment • Considers all possible consequences of the procedure

The nurse is interviewing a patient in the community clinic and gathers the following information about her: She is intermittently homeless and is a single parent with two children who have developmental delays. She has had asthma since she was a teenager. She does not laugh or smile, does not volunteer any information, and at times appears close to tears. She has no support system and does not work. She is experiencing an allostatic load. As a result, which of the following would be present during a complete patient assessment (Select all that apply.)

• Chronic illness • Insomnia • Depression

A 26-year-old patient visits a medical clinic and asks a nurse to provide instruction on how to perform a breast self-examination. "My mom had cancer so I want to learn how. Which domains are required to learn this skill? (Select all that apply.)

• Cognitive domain • Psychomotor domain

Which strategies should a nurse use to facilitate a safe transition of care during a patient's transfer from the hospital to a skilled nursing facility? (Select all that apply.)

• Collaboration between staff members from sending and receiving departments • Using a standardized transfer policy and transfer tool

Which statements reflect the difficulty that can occur for agreement on a common definition of the word quality when it comes to quality of life? (Select all that apply.)

• Community values influence definitions of quality, and they are subject to change over time. • Individual experiences influence perceptions of quality in different ways, making consensus difficult. • Statistical analysis is difficult to apply when the outcome cannot be quantified.

A nurse has been caring for a patient over 2 consecutive days. During that time the patient had an intravenous (IV) catheter in the right forearm. At the end of shift on the second day the nurse inspects the catheter site, observes for redness, and asks whether the patient feels tenderness when the site is palpated. The nurse reviews the medical record from 24 hours ago and finds the catheter site was without redness or tenderness. Which of the activities below reflect the nurse's ability to perform patient evaluation? (Select all that apply.)

• Comparing patient response with previous response • Examining results of clinical data

The nurse applying effective communication skills throughout the nursing process should: (Place the following interventions in the correct order.)

• Complete a nursing history using verbal communication techniques. • Validate health care needs through verbal discussion with the patient. •Provide support through therapeutic communication techniques. • Compare actual and expected patient care outcomes with the patient.

The nurse who works at the local health care agency is transferring a patient to an acute rehabilitation center in another town. To complete the transfer, information from the patient's HR must be printed and faxed to the acute rehabilitation center. Which of the following actions are appropriate for the nurse to take to maintain privacy and confidentiality of the patient's information when faxing this information? (Select all that apply.)

• Confirm that the fax number for the acute rehabilitation center is correct before sending the fax. • Use the encryption feature on the fax machine to encode the information and make it impossible for staff at the acute rehabilitation center to read the information unless they have the encryption key. • After sending the fax, place the information that was printed out in a secure canister marked for shredding.

A nurse admits a 32-year-old patient for treatment of acute asthma. The patient has labored breathing, a respiratory rate of 28 per minute, and lung sounds with bilateral wheezing. The nurse makes the patient comfortable and starts an ordered intravenous infusion to administer medication that will relax the patient's airways. The patient tells the nurse after the first medication infusion, "I feel as if I can breathe better." The nurse auscultates the patient's lungs and notes decreased wheezing with a respiratory rate of 22 per minute. Which of the following is an evaluative measure? (Select all that apply)

• Counting respirations per minute • Asking the patient to describe how his breathing feels • Auscultating lung sounds

A nurse reviews data gathered regarding a patient's response to a diagnosis of cancer. The nurse notes that the patient is restless, avoids eye contact, has increased blood pressure, and expresses a sense of helplessness. The nurse compares the pattern of assessment findings for Anxiety with those of Fear and selects Anxiety as the correct diagnosis. This is an example of the nurse avoiding an error in which of the following? (Select all that apply.)

• Data clustering • Data interpretation • Making a diagnostic statement

A nurse is participating in a health and wellness event at the local community center. A woman approaches with her father and relates that she is worried that her widowed father is becoming more functionally impaired and may need to move in with her. The nurse asks about his ability to complete activities of daily living (ADLs) by asking which of the following questions? (Select all that apply.)

• Describe any problems you have in sitting or getting up from your toilet. • Tell me how often you take a bath during the week and how you bathe.

The nurse is visiting a patient who lives alone at home. While talking with the patient's family caregiver, the nurse learns that the patient has been missing doses of medications. The patient currently self-medicates. The nurse wants to perform interventions to improve the patient's adherence. Which of the following will affect how this nurse will make clinical decisions about how to help the patient improve adherence? (Select all that apply.)

• Determining the value the patient places on taking medications • Determining patient's perceptions of consequences associated with missing specific medicines

A patient has been laid off from his construction job and has many unpaid bills. He is going through a divorce from his marriage of 15 years and has been praying daily to help him through this difficult time. He does not have a primary health care provider because he has never really been sick, and his parents never took him to a physician when he was a child. Which external variables influence the patient's health practices? (Select all that apply.)

• Difficulty paying his bills • Family practice of not routinely seeing a health care provider

A nurse is completing an assessment on a 27-year-old female patient, Which questions best assess the psychosocial aspects of this young woman's health? (Select all that apply.)

• Do you feel safe in your home and at work? • Describe your relationship with your family. • What are your long-term career goals?

A nurse assesses a 42-year-old woman at a health clinic. The woman is married and lives in a condo with her husband. She reports having frequent voiding and pain when she urinates. The nurse asks whether she has to go to the bathroom at night, and the patient re-sponds, "Yes, usually twice or more." The patient had an episode of diarrhea 1 week ago. She weighs 136 kg (300 lb) and reports having difficulty cleansing herself after voiding or passing stool. Which of the following demonstrate assessment findings that cluster to indicate the nursing diagnosis Impaired Urination. (Select all that apply.)

• Dysuria • Nocturia • Frequent voiding

A 48-year-old patient visiting the primary health clinic presents with fatigue and recent weight loss. It is the patient's first visit to the clinic in 2 years. The patient has a family history of cancer and heart disease. During assessment, the nurse finds that the patient has two jobs and works about 12 to 14 hours every weekday. The patient drinks three or four alcoholic drinks a day, relies on fast foods or prepared foods at work, sleeps only 4 to 6 hours a day, and exercises infrequently. The patient takes a nonsteroidal antinflammatory drug (NSAID) daily for knee pain. Which of the following places this patient at risk for development of a chronic illness later in life? (Select all that apply.)

• Eating high-calorie and fatty foods • Exercising infrequently • Excessive alcohol consumption

Which actions implemented by the nurse help grieving families? (Select all that apply.)

• Encourage involvement in nonthreatening group social activities. • Follow up with the family to make sure all their questions are answered. • Remind them that feelings of sadness or pain can return around anniversaries. • Encourage survivors to ask for help. • Look for overuse of alcohol, sleeping aids, or street drugs.

A nurse has the responsibility of managing a patient's postmortem care. What is the proper order for completing postmortem care

• Ensure that the request for organ/tissue donation and/or autopsy was completed. • Elevate the head of the bed. • Collect any needed specimens. • Speak to the family members about their possible participation. • Notify support person (e.g., spiritual care provider, bereavement specialist) for the family. • Remove all tubes and indwelling lines. • Bathe the body of the deceased. • Position the body for family viewing.

A patient is in skeletal traction and has a plaster cast due to a fractured femur. The patient experiences decreased sensation and a cold feeling in the toes of the affected leg. The nurse observes that the patient's toes have become pale and cold but forgets to document and report this to the health care provider because one of the nurse's other patients experienced cardiac arrest at the same time. Two days later the patient in skeletal traction has an elevated tem-perature, and he is prepared for surgery to amputate the leg below the knee. Which of the following statements regarding a breach of duty apply to this situation? (Select all that apply.)

• Failure to document a change in assessment data • Failure to notify a health care provider about a change in the patient's condition

A family is facing job loss of the father, who is the major wage earner, and relocation to a new city where there is a new job. The children will have to switch schools, and his wife will have to resign from the job she enjoys. Which of the following contribute to this family's hardiness?

• Family meetings • Established family roles • Willingness to change in time of stress

At 1300 the registered nurse (RN) says to the assistive personnel (AP), "You did a good job transferring Mr. Harvey into his chair this morning at 0830. I saw that you recorded how long he stayed in his chair. I saw that Mr. Harvey did not have any shoes or nonslip slippers on, though. For safety, the next time you transfer a patient into a chair, you need to make sure that the patient wears slippers or shoes. Please get Mr. Harvey up in his chair again by 1500." Which characteristics of positive feedback did the RN use when talking to the AP? (Select all that apply.)

• Feedback focuses on one issue. • Feedback offers concrete details. • Feedback identifies ways to improve. • Feedback focuses on changeable things

A 9-year-old child has a difficult time making friends at school and being chosen to play on teams. He also has trouble completing his homework and, as a result, receives little positive feedback from his parents or teacher. According to Erikson's theory, failure at this stage of development results in: (Select all that apply.)

• Feelings of inadequacy. • Feelings of inferiority.

The nurse is caring for a patient who is suffering from post-traumatic stress disorder (PTSD) after a motor vehicle accident. What assessment findings reported by the patient provide cues to validate the diagnosis of PTSD? (Select all that apply.)

• Frequent feelings of anxiety • Frequent nightmares • Flashbacks to the accident • Feelings of sadness

A nurse is assigned to a new patient admitted to the medical unit. The nurse collects a nursing history and interviews the patient. Place the following steps for making a nursing diagnosis in the correct order.

• Gather thorough patient data about the patient's health problem. • Review assessment findings, noting objective and subjective clinical cues. • Cluster cues that form a pattern. • Consider whether data are expected or unexpected based on the patient's problem. • Consider the context of patient's health problem and select a related factor. • Identify the nursing diagnosis.

An 80-year-old female patient is admitted to the hospital through the emergency room with acute chest pain. Initial testing shows that the patient had a myocardial infarction. Her current vital signs are BP 156/90; HR 88; respirations 20; she is febrile. The patient has osteoarthritis, causing chronic pain in both knees. She also has glaucoma. The patient is Bosnian and has been in the United States for 3 years. She requires a professional interpreter at bedside when explanations of treatment are provided. Which of the following factors might require you to adapt the way you assist this patient with ambulation? (Select all that apply.)

• Had a recent myocardial infarction • Osteoarthritis of both knees • Needs an interpreter

Two nurses are revising a self-management education program to help patients better manage their asthma. What strategies are most important for them to include in the program? (Select all that apply.)

• Have patients list the medications they are prescribed to take and describe any problems they are having with their medications. • Provide information on how to balance activities during the day. • Ask patients to discuss how other people in their family react to them now that they have asthma.

A nurse is caring for a patient who has just transferred from an intensive care unit and still has a long hospitalization ahead. Because of the complexity of care and the number of caregivers for this patient, the family members feel disconnected from their loved one and not part of the care team. Which interventions would demonstrate care by helping family members? (Select all that apply.)

• Helping family become active participants in care • Providing the family opportunities to discuss their concerns with the health care team • Planning uninterrupted time for the family and patient to be together

Which of the following statements indicate that the new nursing graduate understands ways to remain involved professionally? (Select all that apply.)

• I am thinking about joining the health committee at my church." • I need to read newspapers, watch news broadcasts, and search the Internet for information related to health." • I will join nursing committees at the hospital after I have completed orientation and better understand the issues affecting nursing."

The nurse is evaluating how well a patient newly diagnosed with multiple sclerosis and psychomotor impairment is coping. Which statements indicate that the patient is beginning to cope with the diagnosis? (Select all that apply.)

• I'm going to let the occupational therapist assess my home to improve efficiency." •I'm going to attend a support group to learn more about multiple sclerosis."

Which of the following are strategies for creating work environments that support nurse caring interventions? (Select all that apply.)

• Improving flexibility for scheduling • Providing opportunities to discuss care •Promoting autonomy of practice

A nurse is providing health promotion information at a health fair for female patients who are diagnosed with cancer. What information should the nurse include? (Select all that apply.)

• Information from the local health department about smoking-cessation classes • Strategies to talk with family and friends about the cancer diag-nonis and the side effects from their treatment • How nutritional needs may change based on the diagnosis of cancer and its treatment

A nurse is completing an assessment on a male patient, age 24. Following the assessment, the nurse notes that his family history is not significant for chronic illnesses, and his physical and laboratory findings are within normal limits. Because of these findings, nursing interventions are directed toward activities related to which of the following? (Select all that apply.)

• Instructing him in health promotion activities • Instructing him about routine screenings • Instructing him about proper vaccinations

Which of the following are symptoms of secondary traumatic stress and burnout that commonly affect nurses? (Select all that apply.)

• Lack of interest in exercise; • Difficulty falling asleep; • Lack of desire to go to work; • Anxiety while working

The nurse remains in the room quietly listening. What caring behaviors is the nurse demonstrating? (Select all that apply.)

• Listening • presence

The nurse is assessing an older adult who is grieving after the loss of a spouse. What are normal signs of grief that the nurse would expect to find? (Select all that apply)

• Loss of interest in attending outside activities • Feeling fatigued • Difficulty making decisions • Problems with remembering things • Change in appetite and eating patterns

A nurse who has worked on a cardiac step-down unit for 4 years receives a new patient from the intensive care unit following a 3-day stay for postmyocardial infarction (MI). The nurse knows that the patient is at risk for a recurrent MI, heart failure, arrhythmias, or a stroke. The patient has stable vital signs when arriving at the new unit: pulse 82 and regular, blood pressure (BP) between 120 and 130 over 80 to 90, and denial of chest pain. The nurse checks the patient's IV for proper function, positions the patient comfortably, and explains that the patient will be checked frequently. Sixty minutes after the initial assessment, the nurse responds to the patient's call light. The patient states, "I am not feeling well" Which of the following are evaluative measures the nurse should perform in this situation?

• Measurement of BP • Asking patient to self-report location and severity of pain • Measurement of heart rate rhythm • Asking patient, "Tell me what you mean by not feeling well

A 35-year-old woman has Medicaid coverage for herself and two young children. She missed an appointment at the local health clinic to get an annual mammogram because she has no transpor-tation. She gets the annual screening because her mother had breast cancer. Which of the following are social determinants of this woman's health? (Select all that apply.)

• Medicaid insurance • Lack of transportation • Woman's age

When assessing the health-related knowledge base of both the middle-age patient with an illness and the family, the assessment should include which of the following? (Select all that apply.)

• Medical course of the illness • Prognosis for the patient • Coping mechanisms of the patient and family • Need for community and social services

A school nurse is planning a health fair for children in first, second, and third grades to promote healthy behaviors. The most appropriate health screening for this age-group would be: (Select all that apply,)

• Taking the children's blood pressure • Recording the children's height and weight on a growth chart

A patient is receiving palliative care for symptom management related to anxiety and pain. A family member asks whether the patient is dying and now in "hospice." What does the nurse tell the family member about palliative care? (Select all that apply.)

• Palliative care is for any patient, at any time, with any disease, in any setting. • Palliative care relieves the symptoms of illness and treatment.

A nurse is caring for a 58-year-old patient who had a stroke, causing loss of function of the left leg and reduced movement in the left hand and arm. The patient is alert and able to follow instructions. The patient's dominant side is left. While observing the patient attempt to eat, the nurse notes the patient is unable to use utensils easily in the right hand. The patient is able to lift a glass with the right hand but has trouble opening food containers. Because of weakness on the left side, the patient has limitations in chewing. The nurse identifies a nursing diagnosis of Self Care Deficit: Impaired Self Feeding. Which of the following are appropriate nurse-sensitive outcomes for this patient? ( select all that apply)

• Patient opens food container • Patient gets food onto utensil • Patient able to chew food

A nurse initiates a brief interview with a patient who has come to the medical clinic because of self-reported hoarseness, sore throat, and chest congestion. The nurse observes that the patient has a slumped posture and is using intercostal muscles to breathe. The nurse auscul-tates the patient's lungs and hears crackles in the left lower lobe. The patient's respiratory rate is 22 breaths/min compared with an average of 16 breaths/min during previous clinic visits. The patient tells the nurse, "It's hard for me to get a breath." Which of the following data sets are examples of subjective data? (Select all that apply.)

• Patient statement, "It's hard for me to get a breath" • Patient report of sore throat and hoarseness

A nurse conducts an assessment of a 42-year-old woman at a health clinic. The woman is married and lives in an apartment with her husband. She reports having frequent voiding and pain when she passes urine. The nurse asks whether she has to go to the bathroom at night, and the patient responds,' "Yes." The patient had an episode of diarrhea 1 week ago. She weighs 136 kg (300 lb). The nurse documents the assessment findings listed below. Which of the assessment findings require priority follow-up by the nurse? (Select all that apply.)

• Patient urinates at night. • Patient reports having difficulty cleansing herself after voiding or passing stool. • Body temperature 38°C (100.4°F) • Recent history of weight gain • Knowledge of perineal care • Frequency of diarrhea

The nurse plans care for a 16-year-old male, taking into consideration that stressors experienced most by adolescents include which of the following? (Select all that apply.)

• Physical appearance and body image • Accepting one's personal identity • Separation from family • Taking tests in school

Setting priorities for a patient's nursing diagnoses or health prob. lems is an important step in planning patient care. Which of the following statements describe elements to consider in planning care? (Select all that apply.)

• Priority setting establishes a preferential order for nursing interventions. • Recognition of symptom patterns helps in understanding when to plan interventions.

Health care agencies must provide which of the following based on federal civil rights laws? (Select all that apply.)

• Provide language assistance services at all points of contact free of charge. • Provide auxiliary aids and services, such as interpreters, note takers, and computer-aided transcription services. • Ensure that interpreters are competent in medical terminology. •Provide language assistance to all patients who speak limited English or are deaf.

A 36-year-old patient newly diagnosed with type 1 diabetes shares with you that he is frustrated with the time it takes to prepare meals and monitor his blood sugar. He also is having trouble understanding his insulin schedule. Which of the following suggestions would be most appropriate? (Select all that apply.)

• Provide patient education materials that are easy to read. • Refer this patient to a diabetes support group. • Suggest that the patient make an appointment with a registered dietitian. • Suggest ways to modify his schedule.

A home health nurse is caring for a 48-year-old woman with newly diagnosed diabetes mellitus type 2. She is prescribed to take Metformin IR three times a day. The woman lives alone but has a sister who lives just 5 miles away. The doctor orders the patient to measure her blood glucose daily for the next month. Which of the following show the nurse's ability to minimize common barriers to change as she helps this patient manage her diabetes? (Select all that apply.)

• Provides a simple-language brochure that explains the long-term implications of diabetes type 2 • Informs the patient about availability of a support group in her community • Recommends that the patient's sister be involved in the discussion about medication and glucose monitoring

When planning care for a dying patient, which interventions undertaken by the nurse promote the patient's dignity? (Select all that apply.)

• Providing respect • Viewing the patient as a whole • Showing interest • Being present

A nurse in a community health clinic has been caring for a young female teenager with Type 1 diabetes mellitus for several months. The expected outcome for this patient is to achieve self-management of insulin administration by discharge. Identify appropriate evaluative measures for self-management for this patient. (Select all that apply.)

• Quality of life • Clinic follow-up visits • Adherence to self-administration of insulin • Description of side effects of medications

The nurse is using the QSEN competency of EBP when working with the unit council to initiate a change related to pain management. Which behaviors demonstrate that the nurse is practicing behaviors associated with EBP? (Select all that apply.)

• Reading original research related to pain management. • Valuing EBP as critical to the development of pain management guidelines for the unit. • Describing to the unit council reliable sources for locating clinical guidelines.

Which of the following approaches are recommended when gathering assessment data from an 82-year-old male patient entering a primary care clinic for the first time ?

• Recognize normal changes associated with aging. • Lean forward and smile as you pose questions. •Allow for pauses as patient tells his story.

The following are steps in the process to help resolve an ethical problem. What is the best order of these steps to achieve resolution?

• Recognize that the problem requires ethics. • Gather all relevant information regarding the clinical, social, and spiritual aspects of the problem. • Take time to clarify values and identify the ethical elements, such as principles and key relationships involved. • Articulate a statement of the problem or dilemma that you are trying to resolve. • List all the possible actions that could be taken to resolve the problem. • Develop and implement a plan to address the problem.

A nurse sees a 76-year-old woman in the outpatient clinic. The patient states that she recently started to notice a glare in the lights at home. Her vision is blurred, and she is unable to play cards with her friends, read, or do her needlework. Which of the following nursing interventions are appropriate? (Select all that apply.)

• Refer her to an ophthalmologist. • Suggest large-print books and playing cards.) • Assess her home environment for safety.

A crisis intervention nurse is working with a mother whose child with Down syndrome is hospitalized with pneumonia and who has lost her child's disability payment while the child is hospitalized. The mother worries that her daughter will fall behind in her classes during hospitalization. Which strategies are effective in helping this mother cope with these stressors? (Select all that apply.)

• Referral to social service process reestablishing the child's disability payment • Coordinating hospital-based and home-based schooling with the child's teacher • Teaching the mother signs and symptoms of a respiratory tract infection

A daughter is beginning to assume caregiver responsibility for her 90-year-old widowed father. Her father has hypertension, coronary artery disease, and type 2 diabetes mellitus. Home health services are set for once a week. During the first visit, the daughter expresses concern about all the medications that her father has been prescribed by different doctors and that he has obtained from different pharmacies. The daughter states that her father cannot really tell her what each medication is for or when he should take them. From this initial information the nurse suspects polypharmacy. What medication assessment data are needed? (Select all that apply.)

• Review all medication prescriptions • Match medication prescriptions with the patient's medication • Identify involvement of the caregiver in helping with medication administration • Obtain a listing of any over-the-counter medications

Which of the following actions, if performed by an RN, could result in both criminal and administrative law sanctions against the nurse? (Select all that apply.)

• Reviewing the EHR of a family member who is a patient in the same hospital on a different unit • Applying physical restraints without a written order

A nurse asks an AP to help the patient in Room 418 walk to the bathroom right now. The nurse tells the AP that the patient needs the assistance of one person and the use of a walker. The nurse also tells the AP that the patient's oxygen can be removed while he goes to the bathroom but to make sure that when it is put back on the flowmeter is still at 2 L. The nurse also instructs the AP to make sure the side rails are up and the bed alarm is reset after the patient gets back in bed. Which of the following components of the "Five Rights of Delegation" were used by the nurse? (Select all that apply.)

• Right task • Right circumstance • Right person • Right directions and communication

Which social media uses can be implemented with patients and families without violating confidentiality? (Select all that apply.)

• Social media can be used to provide supportive information. • Family and friends who cannot be present can connect with the patient.

A nurse needs to use clinical judgment before planning and implementing any touch interventions. What does the nurse need to understand about the use of touch? (Select all that apply.)

• Some cultures may have specific restrictions about non-skill-based touch. • Touch is a form of nonverbal communication • Touch can successfully influence a patient's level of comfort.

A nurse is teaching an older-adult patient about ways to detect a melanoma. Which of the following are age-appropriate teaching techniques for this patient? (Select all that that apply

• Speak in a low tone. • Begin and end the session with the most important information • Provide specific information in frequent, small amounts.

A patient diagnosed with colon cancer has been receiving chemotherapy for six weeks. The patient visits the outpatient infusion center twice a week for infusions. The nurse assigned to the patient is having difficulty accessing the patient's intravenous port used to administer the chemotherapy. Despite attempts to flush the port, it is obstructed. This occurred two weeks earlier as well. What steps should the nurse follow to make a consultation with a member of the IV infusion team? (Select all that apply.)

• Specifically identify the problem of port obstruction and attempt to flush the port to resolve the problem. • Explain to the IV nurse the frequency in which this port has obstructed in the past. • Describe to the IV nurse the type and condition of the port currently in use.

A nurse is assigned to care for six patients at the beginning of the night shift. The nurse learns that the floor will be short by one registered nurse (RN) because one of the nurses called in sick. Assistive personnel (AP) from another area is coming to the nursing unit to assist. Because the unit requires hourly rounds on all pa-tients, the nurse begins to make rounds on a patient who recently asked for a pain medication. The nurse is interrupted by another registered nurse who asks about another patient. Which factors in this nurse's unit environment will affect the ability to set priorities? (Select all that apply.)

• Staffing level • Interruption by staff nurse colleague • Competency of assistive personnel (AP)

A nurse sends a text message to the oncoming nurse that states, "Mr. Kodro in room 3348-1 refused to take his sertraline hydrochloride as ordered this morning because he said he was feeling better." What should the oncoming nurse do?

• Tell the nurse who sent the text that the text is a HIPAA violation. • Inform the nursing supervisor.

The nurse enters a patient's room and finds that the patient was incontinent of liquid stool. Because the patient has recurrent redness in the perineal area, the nurse worries about the risk of the patient developing a pressure injury. The nurse cleanses the patient, inspects the skin, and applies a skin barrier ointment to the perineal area. The nurse consults the wound and ostomy care nurse specialist for recommended skin care measures. Which of the fol lowing correctly describe the nurse's actions? (Select all that apply.

• The call to the wound and ostomy care nurse specialist is an indirect care measure. • The cleansing of the skin is a direct care measure.

A mother and her two children are homeless and enter a free health care clinic. Which statements most likely describe the effects of homelessness on this family?

• The family members may have symptoms of malnutrition, such as anemia. • The children are at higher risk for developing ear infections. • All family members may have mental health issues.

Using the Transtheoretical Model of Change, order the steps that a patient goes through to make a lifestyle change related to physical activity.

• The individual becomes angry when the physician tells him that he needs to increase his activity to lose 30 lbs. • The individual recognizes that he is out of shape when his daughter asks him to walk with her after school. • The individual visits the local running store to purchase walking shoes and obtain advice on a walking plan. • The individual walks 2 to 3 miles, 5 nights a week, with his wife. • Eight months after beginning walking, the individual participates with his wife in a local 5K race.

A 63-year-old patient is retiring from his job at an accounting firm where he was in a management role for the past 20 years. He has been with the same company for 42 years and was a dedicated em-ployee. His wife is a homemaker. She raised their five children, babysits for their grandchildren as needed, and belongs to numerous church committees. What are the major psychosocial concerns for this patient? (Select all that apply.)

• The loss of his work role • How the wife may now expect assistance with household tasks and with babysitting the grandchildren.

A nurse is caring for a 76-year-old female patient in the home setting. She just lost her husband from COVID-19 and has four children who live nearby. The patient was an educator and retired only 2 years ago. The nurse applies knowledge of developmental changes and the nature of loss in older adults when assessing which of the following situations? (Select all that apply.)

• The nature of her relationships with her adult children • The patient's perception of the need for caregiving assistance from the family with activities her husband performed • The impact of her husband's death on her monthly income • The patient's current physical functional status

A new nurse graduate is in orientation on a surgical unit and is being mentored by an experienced nurse. Which action completed by the new nurse graduate requires intervention by the experienced nurse? (Select all that apply.)

• The new nurse stops documenting about a dressing change to take a patient some water. • The new nurse gathered the medications for two different patients at the same time.

A nurse is assigned to five patients, including one who was recently admitted and one returning from a diagnostic procedure. It is currently mealtime. The other three patients are stable, but one has just requested a pain medication. The nurse is working with an assistive personnel. Which of the following are appropriate delegation actions on the part of the nurse? (Select all that apply.)

• The nurse directs the assistive personnel to go to the patient in pain and to reposition and offer comfort measures until the nurse can bring an ordered analgesic to the patient. • The nurse directs the assistive personnel to set up meal trays for patients. • The nurse directs the assistive personnel to assist one of the stable patients to sit up in a chair for his meal.

A nurse works with a patient using therapeutic communication during all phases of the therapeutic relationship. Place the nurse's statements in order according to these phases.

• The nurse is mindful of biases and knowledge in working with the patient with B12 deficiency. • After providing introductions, the nurse defines the scope and purpose of the nurse-patient relationship. • The nurse states, "Let's work on learning injection techniques." • The nurse summarizes progress made during the nursing relationship.

The nurse manager from the surgical unit was awarded the nursing leadership award for practice of transformational leadership Which of the following are characteristics or traits of transfor mational leadership displayed by the award winner? (Seledt de that apply.)

• The nurse manager regularly rounds on staff to gather inpu on unit decisions. • The nurse manager sends thank-you notes to staff in recognition of a job well done. • The nurse manager has an "innovation idea box" to which staff are encouraged to submit ideas for unit improvements.

A nurse is caring for a patient who is expressing pain. The patient has two analgesics ordered prn for pain and has been using cold applications on his surgical site for pain relief. The last time an analgesic was given was 4 hours ago. The patient is scheduled for a physical therapy visit in 2 hours. Which of the following demonstrate good clinical decision making during intervention? (Select all that apply.)

• The nurse reviews the options for pain relief for the patient. • The nurse assesses whether the prn medication, ordered every 4 to 6 hours and last given 4 hours ago, is effective and whether a new type of medication is needed. • The nurse considers how the patient might react if the pain medication is held until an hour before physical therapy.

A nurse prepares to contact a patient's health care provider about a change in the patients condition. Put the following statements in the correct order using SBAR (Situation, Background, Assessment, and Recommendation) communication.

• The patient started complaining of nausea yesterday evening and has vomited several times during the night." • "She is a 53-year-old female who was admitted 2 days ago with pneumonia and was started on Levaquin at 5 pm yesterday. She complains of a poor appetite." • "The patient reported feeling very nauseated after her dose of Levaquin an hour ago." • "Is it possible to make a change in antibiotics, or could we give her a nutritional supplement before her medication?"

A nurse who works in an outpatient chemotherapy infusion center is assigned to the care of a 56-year-old male patient who is receiving chemotherapy for colon cancer. This is the patient's first clinic visit. The nurse reviews the patient's medical record and sees a note about the patient receiving instruction on how chemotherapy treats can-cer, but the note does not summarize the patient's response. The doctor makes a quick visit and tells the patient, "We will get your treatment started today and we will be checking your blood each week for any problems. The doctor leaves and the patient asks the nurse, "What are the blood tests for?" To determine this patient's learning needs, what should the nurse assess? (Select all that apply.)

• The patient's behavior as the nurse interacts with him • The patient's level of knowledge about chemotherapy effects • The patient's health literacy • The patient's self-description of severity of his cancer

A 45-year-old woman who is obese tells a nurse that she wants to lose weight. Which assessment findings may be contributing factors to the woman's obesity? (Select all that apply.)

• The woman works in an executive position that is very demanding. • The woman says that she has little time to prepare meals at home and eats out at least four nights a week.

JoAnn, a nurse, is calling a patient's health care provider about a problem that her patient, Ms. Ducote, is having during a blood transfusion. The health care agency uses the SBAR system in reporting patient problems. Put the statements in the correct order according to the SBAR system.

• This is JoAnn. I am caring for Ms. Ducote. She is having labored breathing and her heart rate is higher now than it was an hour ago. She is receiving the second unit of her blood transfusion. She says she is having trouble catching her breath. • Ms. Ducote had surgery earlier yesterday to remove a tumor in the colon. Her hemoglobin was 9.6 grams/dL, and her hematocrit was 33.6% this morning. Her first ordered unit of packed red blood cells infused over 90 minutes, and she is 30 minutes into receiving her second unit of blood now. She states she takes furosemide 20 mg every morning at home. She does not currently have an order for furosemide. • The patient denies pain, and her vital signs are as follows: B/P 150/98 mm Hg; pulse 118; respiratory rate 28; temperature 98.8° F (37.1° C). I think she is showing signs of fluid volume overload. • I think she might need a diuretic ordered. Is it possible for you to come see her soon? Is there anything else you would like for me to do right now?

A patient has gone through a number of treatment changes during a shift of care. During the hand-off report, the nurse plans to communicate effectively with the nurse who will be caring next for the patient for which of the following reasons? (Select all that apply.)

• To reduce the risk of errors to the patient • To provide an optimum level of patient care • To improve patient outcomes

A nurse is providing education to a patient with type 2 diabetes mellitus. Which characteristics does the nurse include in her teaching to explain why type 2 diabetes mellitus is considered a chronic disease? (Select all that apply.)

• Type 2 diabetes mellitus lasts throughout a person's life. • People with type 2 diabetes mellitus have to modify some of their daily activities. • People with type 2 diabetes mellitus require ongoing medical care.

A home health nurse is visiting a 62-year-old Hispanic woman diagnosed with type 2 adult-onset diabetes mellitus following a 2-day stay at a local hospital. The physician ordered home health with placement of the patient on a diabetic protocol for education about diabetes mellitus and a new medication and diet counseling. The patient lives with her 73-year-old husband, who has progressive dementia. Their daughter checks on her parents daily, buys grocer-ies, and helps with home maintenance. The nurse conducts an initial history to gather information about the patient's condition Which of the following data cues combine to reveal a possible health problem? (Select all that apply.)

• Unable to describe diabetes • Has limited health literacy • Patient unable to identify food sources on prescribed diet • Patient has reduced vision and wears glasses

You are a nurse who is working in an agency that has recently implemented an EHR. Which of the following are acceptable practices for maintaining the security and confidentiality of HR information? (Select all that apply.)

• Using a strong password and changing your password frequently according to agency policy • Ensuring that work lists (and any other data that must be printed from the EHR) are protected throughout the shift and disposed of in a locked receptacle designated for documents that are to be shredded when no longer needed • Ensuring that the patient information that is displayed on the computer monitor that you are using is not visible to visitors and other health care providers who are not involved in that patient's care

A nursing student is providing a hand-off report to a registered nurse (RN) who is assuming the patient's care at the end of the clinical day. The student states, "The patient had a good day. His intravenous (IV) fluid is infusing at 124 mL/hr with D›½NS infusing in left forearm. The IV site is intact, and no complaints of tenderness. I ambulated him twice during the shift; he walked to the visitors lounge and back with no shortness of breath, respirations 14, heart rate 88 after returning to chair. He uses his walker without difficulty, gait normal. The patient ate ¾ of his dinner with no gastrointestinal complaints." Which expected outcomes aimed at improving the patient's activity tolerance were discussed in the hand-off? (Select all that apply.)

• Walked to visitors lounge • No shortness of breath • Vital signs after ambulation

A 78-year-old male is going through preoperative screening and preparation for his surgery in 1 week. His wife is with him, and initially they both appear anxious. He will be undergoing a colon resection for colon cancer and will have a permanent colostomy. The nurse knows the patient's age and developmental status will affect how the assessment is conducted. The nurse wants to provide a database that will be useful to the nurses in the hospital and therefore applies clinical judgment appropriately through which of the following: (Select all that apply.)

• When the patient has difficult remembering current medica-tions, the nurse asks the wife to supplement information. • The nurse anticipates the effect of the colostomy on functioning and asks the patient how he perceives life with a colostomy.

When assessing an older woman who is recently widowed, the nurse suspects that the woman is experiencing a developmental crisis. Which questions provide information about the impact of this crisis? (Select all that apply.)

• With whom do you talk on a routine basis? • What do you do when you feel lonely? • Have you experienced any changes in lifestyle habits, such as sleeping, eating, smoking, or drinking?

According to Piaget's cognitive theory, a 12-year-old child is most likely to engage in which of the following activities? (Select all that apply.)

• Writing a story about a clown who wants to leave the circus • Hanging out with a best friend

A nurse performs the flowing four steps in delegating a task to an AP. Place the steps in the correct order of appropriate delegation

• in the next 30 minutes please assist Mr. Malone in Room 418 with his afternoon walk. • Before you take him for his walk to the end of the hallway and back, please take and record his pulse rate. • I will make sure that I check with you in about 40 minutes to see how the patient did. • Do you have any questions about walking with Mr. Malone?

The ethics of care suggests that ethical dilemmas can best be solved by attention to relationships. How does this differ from other approaches to ethical problems? (Select all that apply.) Ethics of care:

• pays attention to the context in which caring occurs. • requires understanding the relationships between involved parties. • considers the decision maker's relationships with other involved parties.


Related study sets

Funds Exam 5 Ch 7, 31, 33, 34 ; Cardio and Respiratory Assessment

View Set

Practice "sometimes, always, never"

View Set

7.5 ~ Linux Application Management

View Set

Domain #3 - Immediate and Emergency Care

View Set

Neuro Review Q's - Alicia Review 11/6

View Set

Ch 13 Special Senses Eyes and Ears

View Set